WILLITST - Will it ever stop

When Bob was in library in University of Warsaw he saw on one of facades caption :"Will it ever stop?" and below some mysterious code:

while n > 1
  if n mod 2 = 0 then
    n:=n/2
  else
    n:=3*n+3

Help him finding it out !

Input

In first line one number n<=10^14.

Output

Print "TAK" if program will stop, otherwise print "NIE"

Example

Input:
4

Output:
TAK

Added by:Krzysztof Lewko
Date:2011-11-09
Time limit:0.906s
Source limit:50000B
Memory limit:1536MB
Cluster: Cube (Intel G860)
Languages:All except: ASM64
Resource:AMPPZ 2011

hide comments
2015-08-29 13:53:04
I spent 4-5 hrs , the key is finding the number is 2^ or not .I used Java long .
2015-08-28 04:48:00 Amanpreet Singh
4 wrong answer due to silly mistake ... i was printing TAk instead of TAK...very easy ques.
2015-08-25 19:43:43 Suvrat Krishna Mishra
finally...use bitwise!!!
2015-08-19 18:13:10 poojan
hardest problem on spoj! it took 1 month to solve! lolz...
2015-08-17 09:26:34 Ravi Chandra
Easy one.AC in one go
2015-08-04 21:19:26 ROHIT Kumar
easy one just try the to make output upto 17 numbers u will get it....
my 50th on spoj....
AC in one go
2015-06-23 17:23:26
wow.. that was easy :p AC in first attemp
2015-06-17 19:19:57 chin
AC in first attempt !!!...:)
2015-06-16 22:37:36 Dipti Singhal
Give heed that program doesn't have to scan number of test cases.
2015-06-16 22:24:53 Gaurav Agarwal
getting WA on 16th test case :<
© Spoj.com. All Rights Reserved. Spoj uses Sphere Engine™ © by Sphere Research Labs.